Radius and Interval of Convergence for (3^n x^n)/(n+1)^2 Series

Click For Summary
The discussion centers on finding the radius and interval of convergence for the series (3^n x^n) / (n+1)^2. The radius of convergence is determined to be 1/3 using the ratio test. For the interval of convergence, it is established that the series converges at x = 1/3 due to the p-series test. At x = -1/3, convergence is confirmed through the alternating series test, which considers the absolute values of the terms. Clarification is sought regarding the book's omission of absolute values, leading to confusion about the conditions for convergence.
badtwistoffate
Messages
81
Reaction score
0
Have to find the radius of convergence and interval of convergence,
the series is (3^n x^n ) / (n+1)^2,
did the ratio test and found the radius of convergence to be the 1/3.
now for finding the interval of convergence I plug in -1/3 and 1/3 into x and find out if it converges or not

For 1/3, it converges due to p-series, 2>1.

But for -1/3 I know it converges but can see why? Any help here at this endpoint?
 
Physics news on Phys.org
You could use the alternating series test.

More simply, you can relate the series at -1/3 to the series at 1/3...
 
I don't quite understand, the alternating series only works when An+1< or = to An, and in that series it doesnt, because An is negative and An+1 is postivie?
Could you elaborate and how I would relate it to the series at 1/3?
 
Look at the alternating series test again, it's the absolute values of the terms that are decreasing (and going to zero) while the sign is alternating.


The series at 1/3 is the absolute values of the terms of the series at -1/3, i.e. you've already should that the series at -1/3 is absolutely convergent.
 
iiiiiiiiiii...
my book doesn't say absolute value... so idk. I see what you mean if that's true. Why doesn't my book say that it says just that its decreasing or equal too...
 
Question: A clock's minute hand has length 4 and its hour hand has length 3. What is the distance between the tips at the moment when it is increasing most rapidly?(Putnam Exam Question) Answer: Making assumption that both the hands moves at constant angular velocities, the answer is ## \sqrt{7} .## But don't you think this assumption is somewhat doubtful and wrong?

Similar threads

  • · Replies 2 ·
Replies
2
Views
2K
  • · Replies 4 ·
Replies
4
Views
1K
  • · Replies 7 ·
Replies
7
Views
2K
  • · Replies 1 ·
Replies
1
Views
2K
  • · Replies 3 ·
Replies
3
Views
1K
  • · Replies 2 ·
Replies
2
Views
2K
Replies
2
Views
1K
  • · Replies 11 ·
Replies
11
Views
3K
  • · Replies 2 ·
Replies
2
Views
1K
  • · Replies 2 ·
Replies
2
Views
2K